Finding the Inverse of a Diagonal Matrix in Terms of Eigenvalues

Click For Summary
To find the inverse of a diagonal matrix D in terms of its eigenvalues λi, the inverse is simply a diagonal matrix with entries 1/λi, assuming all λi are non-zero. This means D^−1 can be expressed as a diagonal matrix where each diagonal element is the reciprocal of the corresponding eigenvalue. To prove that H^−1 = UD^−1U†, one can multiply the matrices and verify that the product yields the identity matrix. The existence of the inverse relies on the condition that none of the diagonal entries of D are zero. The discussion emphasizes the straightforward nature of finding inverses for diagonal matrices and the importance of matrix multiplication for verification.
P-Jay1
Messages
32
Reaction score
0
Diagonalizing an N × N matrix H involves writing it as H = UDU† where D is a
diagonal matrix, with diagonal elements equal to the eigenvalues of the matrix H, and U
is a unitary matrix.
We may write:

D=
(λ1 0 0 ... 0)
(0 λ2 0 ... 0)
(0 0 λ3... 0)
(... ... ... ... λn)

Assuming all the eigenvalues are non-zero, how do I find an expression for the inverse matrix
D^−1 in terms of λi?
And how do I prove rove that H^−1 = UD^−1U†?


For the first question I'm assuming the inverse of D is just:

D=
(1/λ1 0 0 ... 0)
(0 1/λ2 0...0)
(0 0 1/λ3...0)
(... ... ... ...1/λn)

How do I find in terms of λi?
 
Physics news on Phys.org
You don't need to "assume" the inverse of a diagonal matrix is the diagonal matrix having the reciprocal of each entry on the diagonal. Simply multiply the matrices and see what you get.

Of course, the inverse exists if and only if none of the diagonal entries is 0. And what you have already is precisely what is meant by "find an expression for the inverse matrix
D^{−1} in terms of λ_i".
 
Last edited by a moderator:
Thanks Ivy. How do I go about doing the second question?
 
Question: A clock's minute hand has length 4 and its hour hand has length 3. What is the distance between the tips at the moment when it is increasing most rapidly?(Putnam Exam Question) Answer: Making assumption that both the hands moves at constant angular velocities, the answer is ## \sqrt{7} .## But don't you think this assumption is somewhat doubtful and wrong?

Similar threads

  • · Replies 8 ·
Replies
8
Views
2K
  • · Replies 2 ·
Replies
2
Views
1K
Replies
4
Views
2K
Replies
9
Views
2K
  • · Replies 14 ·
Replies
14
Views
2K
  • · Replies 5 ·
Replies
5
Views
14K
  • · Replies 4 ·
Replies
4
Views
1K
Replies
2
Views
2K
  • · Replies 3 ·
Replies
3
Views
2K
  • · Replies 7 ·
Replies
7
Views
2K